Help with Epsilon Delta Proof of Multivariable Limit

Click For Summary
The discussion centers on finding the limit of the function f(x,y) = (xy - x) / (x^2 + y^2 - 2y + 1) as (x,y) approaches (0,1). The user initially assumes the limit L is 0, but realizes that the limit does not exist due to differing results when approaching along various paths. A key error identified is the incorrect inequality used in the epsilon-delta proof, which fails when the denominator is less than 1. Additionally, the conversation highlights the flexibility in defining epsilon and delta, emphasizing that they can be related but are not necessarily equal. Understanding these concepts is crucial for correctly applying epsilon-delta proofs in multivariable calculus.
joe5185
Messages
9
Reaction score
1

Homework Statement


Hey guys. I am having a little trouble answering this question. I am teaching myself calc 3 and am a little confused here (and thus can't ask a teacher). I need to find the limit as (x,y) approaches (0,1) of f(x,y) when f(x,y)=(xy-x)/(x^2+y^2-2y+1).

Homework Equations


|f(x,y)-L|<epsilon
0<sqrt((x-a)^2+(y-b)^2))<delta
I made L=0, assumed epsilon>0

The Attempt at a Solution


Looking at the answer I see that the limit does not exist; however when I do the epsilon delta proof I can't see where I went wrong because I keep getting the result that it does :( ? So I attached a picture detailing my argument and I would love for someone to tell me where I went wrong. I chose L in the epsilon delta definition to be 0 because this is what I get when I approach (0,1) along x=0, y=1, and y=x^3+1 . I am aware that the limit does not exist because if you travel along x=y^2-1 you get a value other than zero. However my only concern is why my logic is not correct in the attached image. Thanks a lot! Also if you have tips for doing these epsilon delta proofs I would love to hear them.
 

Attachments

  • argument.png
    argument.png
    9 KB · Views: 2,339
Last edited:
Physics news on Phys.org
You can factor the numerator and get x(y-1) . Meanwhile your denominator factors and you get x^2+(y-1)^2. If you let ## x=\epsilon ## (it approaches zero) and let ## y-1=\Delta ## you then get a simple expression for the limit in terms of ## \epsilon ## and ## \Delta ##. If you let ## \Delta=\alpha \epsilon ## the result depends on ## \alpha ##. Thereby you don't have a single limit that it converges to. And I see your error=your denominator is greater than zero but it is not greater than 1.(top line=your inequality is incorrect.)
 
  • Like
Likes joe5185
thank you so much. I completely follow you here:). Do you mind elaborating on when I can use the technique where you choose what in your expression is delta and what is epsilon? It seems pretty powerful but I just want to make sure when and how to use it. Thanks for the help
 
joe5185 said:
thank you so much. I completely follow you here:). Do you mind elaborating on when I can use the technique where you choose what in your expression is delta and what is epsilon? It seems pretty powerful but I just want to make sure when and how to use it. Thanks for the help
You have two variables, x, y that are approaching a,b respectively. Let ## x-a=\epsilon ## and ## y-b=\Delta ##. The ## \epsilon ## and the ## \Delta ## both approach zero, but there's nothing that says ## \epsilon=\Delta ##. You can let ## \Delta=\alpha \epsilon ##.The constant ## \alpha ## is quite arbitrary. If you could show your expression to give an answer that is independent of ## \alpha ##, then the limit would be what you computed by evaluating the expression with the ## \epsilon ## and ## \Delta ##. Hopefully this is helpful.
 
very helpful! thanks
 
  • Like
Likes Charles Link
joe5185 said:

Homework Statement


Hey guys. I am having a little trouble answering this question. I am teaching myself calc 3 and am a little confused here (and thus can't ask a teacher). I need to find the limit as (x,y) approaches (0,1) of f(x,y) when f(x,y)=(xy-x)/(x^2+y^2-2y+1).

Homework Equations


|f(x,y)-L|<epsilon
0<sqrt((x-a)^2+(y-b)^2))<delta
I made L=0, assumed epsilon>0

The Attempt at a Solution


Looking at the answer I see that the limit does not exist; however when I do the epsilon delta proof I can't see where I went wrong because I keep getting the result that it does :( ? So I attached a picture detailing my argument and I would love for someone to tell me where I went wrong.
...
The following inequality from your picture does not hold if x2 + (y - 1)2 < 1 .
upload_2016-6-19_18-53-34.png
 
Question: A clock's minute hand has length 4 and its hour hand has length 3. What is the distance between the tips at the moment when it is increasing most rapidly?(Putnam Exam Question) Answer: Making assumption that both the hands moves at constant angular velocities, the answer is ## \sqrt{7} .## But don't you think this assumption is somewhat doubtful and wrong?

Similar threads

  • · Replies 31 ·
2
Replies
31
Views
2K
  • · Replies 9 ·
Replies
9
Views
4K
  • · Replies 8 ·
Replies
8
Views
2K
  • · Replies 15 ·
Replies
15
Views
6K
  • · Replies 12 ·
Replies
12
Views
3K
Replies
19
Views
2K
  • · Replies 6 ·
Replies
6
Views
2K
  • · Replies 5 ·
Replies
5
Views
2K
  • · Replies 13 ·
Replies
13
Views
2K
  • · Replies 20 ·
Replies
20
Views
2K